Вы находитесь на странице: 1из 33

Constitutional Law Outline

Separation of Powers Part I: The Relationship Between the Judiciary and the Other Political Branches
Which is better, large or small gov? Jefferson / Anti-Federalist viewpoint / Civic Virtue: Small government is good; we want people to participate! Civic Virtue When people are presented with everyone elses concerns, they dont necessarily vote in their own best interest. Participatory democracy allows for this. Problem: Active participation only promotes civic virtue in a homogenous setting (for example, if half the people are typical and half are Amish, they will have different ideas of the greater good so it might not promote civic virtue) Madison / Federalist viewpoint / Self Interest: Large government is good; we want to avoid having a homogenous faction or to prevent them from realizing they are the majority. Self-Interest People generally only vote for their own self-interest or the interest of their group, not the nation as a whole. We dont want homogenous decision making bodies/factions and we cant ever really have a society without factions because peoples faculties are different and the wealth isnt evenly distributed. A large government will offset this. Judicial Control of the Acts of the Legislature and Executive Judicial Review: In order for the S Ct to hear a case, it must have either original or appellate jx. Is there original jx? Under Article III, Section 2, the S Ct has original jx in all cases affecting ambassadors, other public ministers and consuls and those in which a state shall be a party. According to M v. Ms reading of the exceptions clause, Congress may neither restrict nor enlarge the S Cts original jx. Is there appellate jx? Article III, Section 2 provides that in all the cases listed in Section 2, Clause 1, except those over which the S Ct was given original jx, the S Ct shall have appellate jx, both as to law and fact, with such exceptions, and under such regulations as the Congress shall make. Exceptions Clause Marshall took this section to mean that Congress could remove appellate jx from the Court, not that Congress could add things to the Courts original jx. See discussion below for more. Extent of Judicial Review: Although the Constitution does not explicitly state it, the Court has held that its appellate jx includes the power to hear appeals regarding: (1) Acts of Other Federal Branches (It is emphatically the province and duty of the judicial department to say what the law is. M v. M.) Reasons for Judicial Review of other branches: Fox in the Chicken Coop Concern We dont want Congress to be the body that decides its own power (however, Marshall fails to note that the Court is deciding its own power in this opinion) Institutional Competence: we dont want Congress to have to be its own judge, because Congress can be overly political and prone to the mood swings of the nation Problem: This might negatively affect civic virtue, because the members of Congress wont be required to consider whether or not their actions are Constitutional. They should, however, always consider what is best of the polity, so civic virtue is completely destroyed. Constitution as a Species of Law The role of the Court is to interpret the law, it is their expertise and they have a 1

unique competence in doing so. This assumes that the Constitution is a species of law? One branch needs the final say, otherwise there would be constant squabbling. Clearly the Court is the most qualified to take on this responsibility. Treating the Court this way makes it a co-equal branch, which is more in line with Federalism. The Supremacy Clause Article VI Section 2 This Constitution, and the laws of the US which shall be made in pursuance thereof ... shall be the supreme law of the land. Marshall read the Supremacy Clause as supporting judicial review (pursuant to the Constitution indicates that it is possible for Congress to pass unconstitutional laws). But, this Clause is not about inter-branch disputes, but instead disputes between the Federal Government and the States. (2) Judgments of states courts in cases that fall within federal judicial power. Martin v. Hunters Lessee. Storys Reasoning: It is the type of case, not the court, which determines if the S Ct has jx. Story tried to argue that it is mandatory under Article III that there be some court that has jx over every type of case in the US. The problem is that this has never been true, thanks to Marshalls interpretation of the exceptions clause. Uniformity and Hospitality At the time there were huge cultural differences between slave and non-slave-owning states, and the country didnt need anything else to split it. Sources of Judicial Decisions Text of the Constitution Plain Meaning This is a red flag, if there were a plain meaning the case wouldnt be before the Supreme Court. Common Understanding What do most people understand a word or phrase to mean? Intent of the Framers / Ratifiers What did the framers intend a word or phrase to mean? Problems with this approach: Whos intent, which framer? What does the intent of the framers have to do with people today? Deference to Other Branches The Court wants to respect its co-equal branches. It assumes that Acts of Congress are constitutional, and it takes a lot to get past this. McCulloch v. Maryland When Congress has exceeded its enumerated powers without stepping on any individual toes but instead state toes, the Court will usually let it go because the states are represented in Congress. This is a federalist argument. Also, the degree of necessity is a choice best left to a legislative body, they do the most cost/benefit analysis so they should decide what is necessary. Nature of the Document It is the constitution we are expounding! It must be malleable and meant to endure. Flexibility is necessary so we must allow the political process. Structure of the Document Where is the provision in question found? Is it in a list of powers granted, a list of restrictions, etc.? Precedent Are there previous cases dealing with the issue? Stare Decisis Natural Law This is when the Judiciary is at its weakest! 2

Does Natural Law exist? What if NL is nothing more than a current moral consensus? 1st principles of social K what is this? Universal feature of humanity according to this, and unlike the current moral census, NL shouldnt differ depending on when and where you live the problem with this is: Who knows what this is? If it exists, who should find it? Does the IX Amendment indicate that there are indeed natural rights? Should the judiciary decide what natural rights are protected? Who will regulate the judiciary? But, if the court appeals to NL to void a decision made by the majority, arent they imposing their own moral views on the majority?

Legislative Control of the Judiciary Constitutional Amendment If Congress doesnt like a particular reading of the Constitution, they can amend the Constitution so that it conforms to their view. Power to Appoint The President chooses Justices with similar political compasses, and the Senate must approve them. This is no way guarantees that a Justice will act the way the President hopes. This is the most controversial control race and gender are big issues here. Impeachment Article III lays out how a Justice may be impeached, but no S Ct Justices have yet been kicked out. Justices have a certain amount of political capital, but the more you spend the less you have. So fear of impeachment will still affect the Justices. Limited Tenure Justices are members of the Court until they resign or die, but many state courts have put caps on how old a judge can be. Exceptions Clause of Article III Congress can limit the appellate jx of the Court via the Exceptions Clause. However, there are two restrictions on this: 1) Congress cannot violate an external restraint (i.e. a restraint found outside of the proposed legislation) in order to restrict the Courts jx. I.e. Bill of Rights There must still be a route to the S Ct, which means that Congress cant remove appellate jx and then declare that neither state nor federal courts can hear the case (Boumediene). Boumediene v. Bush Guantanamo prisoners case. Ct. found that even though not US nationals they could bring a habeas petition. Also found that the procedures specified by Congress in the Detainee Treatment Act and Military Commissions Act violated the prisoners habeas rights. 2) If Congress is restricting the Courts ability to hear cases regarding the restriction of a fundamental right, the legislation must have a compelling end and narrowly tailored means (i.e. must be capable of passing strict scrutiny). The Court always will have jurisdiction to consider the constitutionality of legislation depriving it of jurisdiction, but be careful in how you understand the role of strict scrutiny in this context. Those who support the traditional view that Congress power here is plenary (because Congress has the power to restrict jx and to create the inferior federal courts?) argue that Article III itself makes the choice between federal and state courts a compelling interest and that, obviously, legislation choosing state courts in certain cases is narrowly tailored. Counter Argument: This would put too many holes in the Courts jx, and maintaining the separation of powers is a compelling state interest as well because the separation of powers prevents one branch from unduly impeding another in the performance of its essential role in the Constitutional plan. What is the Courts essential role? All three of the roles listed below are relied on to explain why Congress power to restrict jx should be limited, even though Article 3

III gives them so much power. 1. Constitutional Guardian (if this were the essential role, then any restriction of its jx to hear cases arising under the constitution would be wrong) 2. Unification 3. Supremacy Congress Cannot Attempt to Control the Court Via Statutes Statutes are not permissible under the Exceptions Clause if they indirectly affect the Courts jx by telling the Court how to interpret something. US v. Klein Congress held that Presidential pardons were proof of wrongdoing, which defeated the type of case before the court and thus deprive the Ct of jx. The Court noted that a statute is only permissible under the clause if it is a denial of the right to appeal in a particular class of cases.

Federalism: Scope of Power Delegated to Congress


Federalism Definition: The general power to govern is reserved to the individual states, and the federal government is granted only certain enumerated powers. Thus with all acts of Congress, we must first determine whether Congress has been granted the power to so act. 10th Am The powers not delegated to the US by the Constitution, nor prohibited by it to the States, are reserved to the States respectively, or to the people. Why do we like Federalism? Efficiency state governments can more quickly deal with state issues Promotes Individual Choice people who may be a minority nationally might often be the majority on a state/local level; this encourages people to participate in the government process Encourages Experimentation states can experiment with different ideas and we as a country can see what works Promotes Democracy people are more willing to participate in politics when they are close to home Prevents Tyranny the two sides balance and limit each other Judicial Enforcement of Federalism The Constitutions enumeration of congressional powers allocates power between the nation and the states. We generally think that federalism is a good thing, but the question is: should the courts enforce this allocation of power, or should it be left up to political checks on Congress? If Congress wont police itself, then its back to M v. Ms fox in the chicken coop theory federalism must be judicially imposed. If the point of federalism is to prevent tyranny, how can we argue that it is tyrannical what the state representatives in Congress are doing? Maybe tyranny of the majority? Scope of Congresss Powers Legislative Power This is basically the power to makes laws, but it also includes the power to investigate, hear, and consider matters upon which legislation may be enacted, and do all things necessary and proper to the enactment of legislation. Enumerated Powers Article I, Section 1 All legislative powers herein granted shall be vested in the Congress of the United States... Article I, Section 8 Lists the extensive powers of Congress. Implied Powers In addition to those power specifically enumerated in the Constitution, certain broad federal powers 4

are to be implied from the N & P clause. McCulloh v. Maryland N&P Clause: Article I, Section 8, Clause 18 Congress shall have the power to make all laws which shall be necessary and proper for carrying into execution the forgoing powers, and all other powers vested by this constitution in the government of the US, or in any department or office thereof. Appropriate Means: Congress is not limited to means that are absolutely necessary. Instead, it may use any appropriate means to achieve the ends specified in the enumerated powers (i.e. any means not prohibited by the Constitution). The need for the means is for Congress, not the Court, to determine. Limits on Congresss Power 10th Am: The powers not delegated to the US by the Constitution, nor prohibited by it to the States, are reserved to the States respectively, or to the people. Congress cannot commandeer state legislatures, i.e. they cant compel state legislative or regulatory activity, and they cant commandeer the state executive branch. But... Congress can induce state action by putting strings on grants. See the spending power. See the analysis below under 10th Amendment as a Limit The Commerce Clause - Generally Article I, Section 8, Clause 3: Congress has the power to regulate commerce with foreign nations, and among the several states ... The power to regulate includes the power to prohibit as well as the power to encourage activity. Foreign: Only the federal gov can regulate this. Among the States: The federal power to regulate interstate commerce is concurrent with state power over transactions within the state. Stewarts Def of Commerce: Buying and selling of goods and services and the regulation of the routes through which they pass in trade. Congresss Power under the CC: The CC allows Congress to adopt legislation that: (1) regulates the channels of interstate commerce; (2) regulates the instrumentalities of interstate commerce and persons and things in interstate commerce; or (3) regulates (economic) intrastate activities that have a substantial effect on interstate commerce. US v. Lopez Channels & Instrumentalities Congress has plenary power to regulate the channels (interstate roads, waterways, airways, etc) and the instrumentalities (people, machines, and other things that carry things in commerce) (Lopez). This includes the authority to prohibit any goods, persons, or activities found by Congress to be harmful to public health, safety, welfare, or morals. Originally this only mattered if harm was caused in the state of destination, not the state of origination. That is why initially Congress couldnt ban goods made by child labor. Previous Cases that Led to This Gibbons v. Ogden Congress can grant licenses in navigable waterways. Shreveport The ICC was able to control intrastate pricing of the railroads. Darby The Court upheld the Fair Labor Standards Act which set minimum wage, maximum hours, and age restrictions (this overruled Hammer). Lottery Case A ban on intrastate lottery tickets was fine, because a prohibition of transport is a transport regulation. Activities with a Substantial Effect Under the CC, Congress can regulate economic activities that in the aggregate as a class have a substantial relation to interstate commerce, i.e. those activities that substantially affect interstate commerce (Lopez). This is allowed based on the CC in conjuncture with the NPC since by definition no intrastate 5

activity is interstate commerce. Lopez restricts the reach of the necessary and proper clause here to the regulation of intrastate economic activity, since otherwise in light of our integrated national economy, Congress would be able to regulate all activity in clear violation of the intent of federalism. This restriction is similar to the 10th Am enclave previously imposed by the Court, limiting the Cts power under the CC & NPC only to intrastate activities that had a direct effect on interstate activity. Lopez Congress wasnt allowed to impose a gun ban in school zones, because the activity was non-economic. Morrison Violence against women is non-economic, so Congress couldnt pass legislation allowing for a private cause of action to allow women to get into federal courts. Previous Cases that Led to This Wickard v. Filburn A farmer grew more wheat than he was allowed. Even though it was a small amount for home use, the Court reasoned that in the aggregate such farmers substantially affected interstate commerce. Jones & Laughlin Congress could regulate the means of production to prevent unfair labor practices. Perez v. US Small time loan sharker was prosecuted. Heart of Atlanta No discrimination in public accommodation because it decreases travel and affects interstate commerce. Ollies BBQ No discrimination in restaurants; they purchase food from out of state and less guests means less food will be purchased. Gonzalez v. Right Congress can ban the cultivation of medical marijuana as well as nonmedical, because it is impossible to tell what the effect of the medical on the non-medical market is.

Courts Treatment of the CC Historical Approach Pre-Depression / Depression Limits on Congress Power under the CC Direct/Indirect Approach (up to 1937) In regulating commerce, Congress has the power to act via the Commerce Clause (interstate) and the Necessary & Proper Clause (intrastate). But, the Court didnt want Congress to have too much power so it restricted the N&P Clause: The N&P clause can be divvied up between things that have direct and indirect effects on interstate commerce, and Congress has no power over things that cause indirect effects. (This is the 10th Amendment Enclave - Those acts which are not themselves interstate commerce, which nonetheless do have an effect on interstate commerce, but whose effect is indirect, are left to the state to regulate.) How do you determine if something is a direct or indirect affect? It seemed as though you need an intervening act to occur before the harm will take place. For example, in Knight the sugar monopolist would have to choose to raise prices before any harm would occur. But see Houston East & West Congress allowed for the regulation of intrastate highway tolls even though there was no effect on interstate commerce until people chose to use certain lines. Unlike Knight, in Houston there was a bad actor and it was clearly commerce, not production. What counted as Commerce at this time? R: Manufacturing was not part of commerce, it was part of production, and thus Congress couldnt regulate it. (See Knight) But see Stafford The Justice calls the entire process (growth, production, manufacture, sale, etc) the stream of commerce, so concerns about the 10th Amendment enclave disappears because you can go directly to the CC. R: Congress did not have to permit use of interstate transport of goods that causes harm in the state of destination (see Champion v. Ames the interstate sale of lottery tickets was banned). But see Hammer v. Dagenhart Goods produced by children could not be banned, the injury 6

did not occur outside the state. Apparently the court did not think that economic injury mattered here, because clearly goods produced by children would be cheaper, and cheaper prices would drive down other markets and injure other sellers. R: Congress couldnt simply regulate the market, even if it might be in the best interest of the country. This provides Congress with too much power. Schechter Congress tried to regulate work-weeks, wage, child labor, collective bargaining, etc., but the Court thought it was beyond their power. Carter Tried to set minimum prices on coal and it opened up the doors for collective bargaining. The court struck this down as well. Post-Depression An Expansion of Congress Power Under the CC Hands Off Approach (from 1937 to 1995) Enter Jones & Laughlin: The Court abolished the direct/indirect distinction in regards to intrastate commerce, effectively killing the 10th Amendment Enclave. Now, Congress has the power to regulate things that might have a noticeable effect, even if that thing is intrastate commerce. Now the Court looks at things in the aggregate. Is there a gross and substantial effect on interstate commerce? If yes, then Congress can regulate it. (See Wickard Even though the farmer had a local, de minimus affect on the market, there were enough farmers like him that the overall effect on the market was great; Perez he was loan sharking, but on a very small scale, and he was still caught by the CC.) Warning: If something is very local, the court might read an ambiguous statute as not applying to it. I dont think she gave an example. R: Congress can now regulate goods in order to prevent harmful activity from taking place in the state. See Darby, which overturned Dagenhart - The best way of making sure that something is not transported interstate is to prevent it from being made. On the flip, the best way to make sure something doesnt occur to prohibit its interstate transport. This is the bootstrap argument, whose logical conclusion would let Congress to do whatever it wants. The other result of Darby is that Congress can regulate the production/sale of goods that will not be transported interstate. Otherwise one group would be disadvantaged. The affect that an unregulated intrastate market on a regulated interstate market could be huge. Usually today Congress doesnt regulate businesses for example that have less than 25 employees. States regulate these. The Court was willing to stretch the connection between the CC and the regulated activity. But there must be a rational tie between the regulated activity and interstate commerce. However, what is a rational tie? If Congress can come up with something plausible, the Court would normally defer. Heart of Atlanta The Court upheld a statute that disallowed segregation, because segregation meant that fewer black individuals could travel, and less tourism affected interstate commerce. Ollies BBQ Segregation meant that fewer people would eat at a restaurant. Since the restaurant normally bought out of state food stuffs, interstate commerce was affected. Present Restrictions on Congress Power Under the CC Return to Judicially Enforceable Limits Based on Economic/Non-economic (from 1995 on) In US v. Lopez (the gun-free school zone case), the Court seemed to pull back form its forgiving position on the CC and for the first time since the Great Depression it set a limit on the CC. However, NOTHING was overturned! Jones & Laughlin is still good law! The Court held that CC applies to three situations: 1) The use of the channel of interstate commerce 2) The instrumentalities of interstate commerce, or persons or things in interstate commerce, even though the threat may come only from intrastate activities 3) Those activities that in the aggregate as a class have a substantial relation to interstate commerce, i.e. those activities that substantially affect interstate commerce. New Magic Language: The intrastate economic activity in the aggregate of the class 7

affects interstate commerce. Rehnquists arguments against: It would be the death of judicially enforceable federalism if the court allowed the US to win this one. He drew a line between economic and non-economic activity this is similar to the 10th Amendment enclave from before. The jxl nexus you could rest jx on the CC of the NPC, but requires that the gun move via interstate commerce which would be hard to legislate around. The link between the regulated activity and interstate commerce was too attenuated. You have to pile inference upon inference to convert congressional authority to general police power. Basically, for want of a nail, a shoe was lost; for want of a shoe... reasoning. Concurring/Dissenting Opinions: Kennedy/OConner Worried about traditional areas of state concern and fostering accountability. This argument presupposes silent politicians, which is not the case. Thomas Wanted to return to the old understanding of commerce that had nothing to do with agriculture, production, or aggregation. Congress would be able to regulate interstate commerce and intrastate commerce that had an effect on interstate commerce. Breyer Dissented, arguing that the nature of the intrastate activity did not matter, only its impact on interstate commerce. Impact alone is enough. Souter Dissented, arguing that the Court should defer to Congress. They are the fact finding and cost analysis body and they know best. US v. Morrison The Violence Against Women Act allowed for civil remedies for victims of gender violence so that women could get into federal courts and avoid state court bias. Gender violence is non-economic, so it doesnt withstand the Lopez test. In order to get from gender violence to interstate commerce, you would have to employ the but for a nail, a shoe was lost type of reasoning. Gonzales v. Right Congress has the authority to regulate the private cultivation and use of medical marijuana, even though the state permitted it. Rationale: Its too hard to differentiate between pot grown for personal use and pot grown for the market and, unlike in Lopez and Morrison, the sale and distribution of pot is clearly economic. State Regulation of Interstate Commerce Gibbons Federal law preempts state law in regards to the commerce clause. Disanto v. Penn Congressional silence was meant to preempt state action Court considered that the state was a bad actor Cooley Regulation of local as opposed to national things is ok. How do you know if its national? If it demands a uniform rule. This is a bad distinction because 1) it is unclear, and 2) federal rules are often not uniform. See handout on Dormant Commerce Clause! The Power to Tax Article I, Section 8, Clause 1 Congress shall have the power to lay and collect taxes ... to pay the debts and provide for the common defense and general welfare of the US. The first question was whether this power could be exercised only in the service of pursing goals enumerated elsewhere in the Constitution or of it served as a more sweeping basis for national regulations. The latter had proved true. Pre-Depression Era: Motive is not really an issue. Instead, the question is can a regulation be justified under its necessity to collect the tax. Tax the object, or tax the income, dont tax those who fail to follow a prescribed course of action. US v. Doremus Upheld taxation on certain drugs; used the taxing power to attack a moral issue. The Court only considered whether the provisions at issue had any relation to the raising of revenue. Thought that the only restriction on Congresss taxing power was national uniformity. 8

Bailey v. Drexel Didnt follow Doremus. Didnt uphold the taxation on goods produced from child labor. Does this law impose a tax with only the incidental restraint and regulation which tax must inevitably involve? Or does it regulate by the use of the so-called tax as a penalty? R: If it is tax as a penalty, it is not valid unless Congress would be able to regulate the activity in the first place. This rule cause problems, because how do you decide whether it is a penalty or a tax? Now: Congress can tax in order to regulate, assuming that Congress could regulate the activity in the first place. Where Congress cant regulate, the tax will be upheld under the taxing power if its dominant intent is revenue-raising (rather than prohibition or regulation). It is the same as saying that the tax will be invalid only if its dominant intent is penal (rather than fiscal). Restrictions: No tax on exports Tax on income used to have to be apportioned, but it no longer is because the 16th amendment made income tax exempt from apportionment Indirect taxes have to uniform across the country The Power to Spend Article I, Section 8, Clause 1 Congress shall have the power to ... pay the debts and provide for the common defense and general welfare of the US. Congress is the best equipped branch to determine what constitutes the general welfare, so the Court will usually defer to their judgment. Conditions Conditions attached to grants are constitutional if either: 1) They are necessary and proper to the spending itself, which requires that they: Be rationally related to the (general welfare) purpose of the spending. This is the Hamilton viewpoint that spending is only limited only by the requirement that it provide for the general welfare. The Court adopted this view point in US v. Butler. It is in contrast to Madisons viewpoint that money could only be spent to meet one of the enumerated goals listed in 8. Be unambiguous so that people/states can decide if they want to accept the money. Grantees must always have the option of accepting the money. In Butler, the Court invalidated a statute arguing that it coerced farmers into joining, although this argument was tenuous and today it probably wouldnt fly. Not be violative of external restraints. Congress cannot condition spending on doing something that would violate a constitutional right that has not been waived (i.e. cant encourage the states to violate the constitution). It is unclear whether or not it is ok if you waive the right And, possibly, relate to how the money is spent. South Dakota v. Dole OConnors Opinion So, for example, in Butler OConnor thinks the Court was wrong not because the condition was not supported by the spending clause + the necessary and proper clause (she agrees it was not so supported) but because the condition, limiting the amount of grain that could be grown, could have been insisted upon by Congress as a result of a properly-understood commerce clause + necessary and proper clause. 2) Congress could impose the condition on the grant recipient directly (i.e. insist that the recipient so what the condition requires under a different grant of authority, usually the commerce clause). The War Power 9

This is a sweeping power, and it doesnt necessarily require that we be at war at the time for Congress to legislate under its authority. Woods v. Cloyd Congress froze the housing market to wartime levels, because the disruption in the housing market was due to the war and the returning veterans. Jacksons Concurrence: Be careful, because we dont want blind reactions to patriotism. During wartime, Congress and the Court are at their most passionate and least objective so this might be abused. Rumsfeld v. FAIR The military can insist on access to schools, because raising an army requires access. Power to Enforce the Reconstruction Amendments Congresss Interpretive Role with the 14th Am The Enabling Clause of 14th Am gives Congress the power to enforce, by appropriate legislation, the provisions of the amendment. Does this mean that Congress has an interpretive role in deciding when state action violates the 14th Am, or should all decisions of constitutionality be left up to the Court? Congress has power to remedy violations of equal protection and due process (Katzenbach), so it seems that Congress plays an interpretive role to an extent. Limitations on Congress Congruence and Proportionality Although Congress has wide latitude in remedying violations of the 14th Am, there must be congruence and proportionality between the injury to be prevented or remedied and the means adopted to that end basically, it must be narrowly tailored. Congruence C between means used and ends to be achieved (ends to be achieved is the remedy). Rationally related? You can only remedy discriminatory intent, not effect, with EP (Equal Protection). What about DP (Due Process)? Morrison remedy of treble damages and sue in fed ct. against perpetrators of sex crimes is NOT congruent because the remedy has nothing to do with prosecution of sex crimes. Doesnt make it more likely to prosecute; only gives victim a new civil claim against the perp in fed ct.. QUESTIONABLE ASSUMPTION: Case mostly deals with how Equal Protection effects states and not individuals. Proportionality Remedy must be tailored to fix violation of the state and not so broad as to sweep in unintentional or benign conduct (of state). Trigger There must be something that triggers the legislation, Congress cant simply change the substantive law (Flores Congress was not permitted to set a new standard of review for fundamental rights violations when no actual violation could be shown). This means that Congress can remedy, but not adopt legislation that expands substantive constitutional rights beyond those defined by the S Ct. Flores Legis must be remedial and preventative not substantive. Intent v. Impact If there is no discriminatory intent, but only discriminatory impact, the state hasnt violated the 14th Am unless there are no legitimate ends and rational means (Washington v. Davis). State Sovereign Immunity and the 14th 14th Amendment postdates the 11th Amendment. Thus, current Court ALSO states that Congress can lift state immunity ONLY when
Congress acts within scope of Section 5 of the 14th Amend. BUT CANNOT lift if acting within scope of Commerce Clause.

10

This is problematic in employment discrimination cases.

If Congress is regulating pursuant to the 14th Am, and not Article 1, it can lift the states sovereign immunity. But, Congress must show that there is a pattern of unconstitutional discrimination by States, and it must meet the congruence and proportionality requirement. Board of Trustees v. Garrett (Title I of ADA if statute goes beyond remedial powers, violates congruence and proportionality, then private suits are not proper and states have immunity from suits. Heightened level of review if dealing with suspect or semi-suspect category, or involving a right that is fundamental for due process or equal protection. ct. is more likely to say in these cases that it was congruent and proportional and thus proper under 5. See Hibbs (FMLA case, trying to redress gender so heightened scrutiny) and Tennesse v. Lane (Title II of ADA equal treatment for state services (voting)). If actual 14th amendment violation then have then Congress has power to give right to recover damages. US v. Georgia (prisoner cant move around his cell in his wheelchair). Because of Seminole Tribe of Florida v. Florida (1996) can no longer use Commerce Clause, only 13th, 14th, or 15th amendment remedial powers. 3 step analysis: 1. Was state conduct an actual violation of plaintiffs constitutional rights. if yes, then clearly can allow them to sue without worrying about whether response is congruent-or-proportional. U.S. v. Georgia 2. Does it involve either a suspect or semi-suspect class or a fundamental right. if yes, apply heightened scrutiny. Hibbs 3. if no to 1 and 2 then can only allow to sue under 5 of the 14th only if Congress found widespread unconstitutional state conduct in the area in question and if found Congress must you narrowly tailored methods to combat the state conduct. Garrett How else can we get around State Sovereign Immunity? States can consent to suits The doctrine does not prohibit suits brought against state officials for injunctive relief as opposed damages The US can still a damages bring suit Waiver of the immunity can be a condition of a grant Standards of Review the higher the standard of review, the more likely it is that the state law will run afoul of the 14th Amendment Rational Basis Review Legitimate interest, rationally related means Disability cases fall into this category. Heighten (Intermediate) Scrutiny Requires and important governmental interest and means that are substantially related to this interest. This includes gender discrimination Strict Scrutiny Compelling ends, narrowly tailored means Fundamental rights violations fit in here. 11

Enforcement of the Reconstruction Amendments Against Individuals The 13th Am abolished slavery and servitude. How does this affect individuals and Congress? Congress: The Civil Rights Cases - The enabling clause gave Congress the power to pass all laws necessary and proper for abolishing all badges and incidents of slavery in the US. Jones v. Alfred H Mayer - Congress has the power rationally to determine what the badges and incidents of slavery are. Individuals: Jones Acts of Congress resting on the 13th Am can prevent private racial discrimination and not just discriminatory laws. How does the 14th Amendment apply to individuals? State Action Doctrine For the 14th Am to be implicated (and therefore for Congress to have power to enforce its protections), there must be either a sufficient degree of state involvement with the action or a failure by the state to act in circumstances where the Constitution affirmatively requires action. Morrison The Court didnt uphold the clause of the VAWA that allowed recovery of compensatory and punitive damages because the clause was not directed at a state but instead at private actors.

The Tenth Amendment as a Limit on Congressional Power


Tenth Amendment The power not delegated to the US by the Constitution, nor prohibited by it to the States, are reserved to the States respectively, or to the people. In the Context of the Treaty Power In regards to statutes passed to implement non-self-executing treaties, the Tenth Amendment alone does not determine whether it was a valid exercise of power. Article II, Section 2, Clause 2 The President shall have power, by and with the advice and consent of the Senate, to make treaties, provided two thirds of the Senate present concur... Article VI, Clause 2 All treaties made, or which shall be made, under the authority of the US, shall be the supreme law of the land... N&P Clause When the treaty is valid, the statute is valid as the necessary and proper means of executing the legitimate power of the federal gov. Missouri v. Holland migration of wild birds is a national problem, best dealt with by a national solution; therefore, no Tenth Amendment rights of individual states will be allowed to stand in the way of such a solution. Treaty power is an independent source of authority. Treaty may not violate constitutional guarantees. A treaty may not violate any distinct constitutional prohibitions or guarantees (e.g. the Bill of Rights) Reid v. Covert woman killed husband on army base and was convicted in military tribunal. But this was not legit since didnt get a fair trial. In the Context of the Commerce Clause Initially, Congress could not use the CC to regulate areas of traditional governmental functions (National League, 1976- Congress could not impose wage/hour rules on state employees. impairs the States integrity or their ability to function effectively in the federal system). This was overturned shortly after because it is too hard to determine what traditional governmental functions are (Garcia, 1985 mass transit employees). It was too subjective. The Court thought that states were still protected because they are all represented in Congress. Now, Court has held that the federal government may not: (1) compel a state to enact or enforce a particular law or type of law; or (2) compel state/local officials to perform federally specific 12

administrative tasks. Holding (1): NY v. US radioactive waste case: state attempted to force each state to make its own arrangements for disposing of the low level radioactive waste. If didnt would have to take title to waste and be liable for damages in connection. Cannot commandeer the legislative process of states. Holding (2): Printz v. US law ordered local law enforcement officials to conduct background checks on prospective gun purchasers until national computerized system could be phased in. When is Congress commandeering, when is it not? Acts that regulate generally (laws of general applicability) are ok but Acts that are directed solely at the states are not. It seems like this opinion answers the question of what is a traditional governmental function?, so has Garcia been overturned? Its unclear. I think no but def limited the Scope. (Figure out if this is from NY v. US or Garica?) While the feds cannot compel states to enforce federal policy, they can provide incentives. See the spending power. If an Act does not require States to act in their sovereign capacity or to enact legislation, it is ok (Reno v. Condon States not allowed to sell personal info n the records of state motor vehicle departments. Regulates states as the owners of the databases. Doesnt require them to regulate their own citizens. Not required to enact laws or regulations and doesnt require state officials to assist with enforcement. Also was generally applicable and did not regulate states exclusively because it regulates the universe of suppliers in the market). Reno Rule from E & J Even if the primary entity affected is the state, it is still ok. It goes to intent if intent is general applicability then its ok. Overall Law of general applicability Ok with respect to interstate commerce and intrastate commerce with economic activity with substantial affect in the aggregate Ok with respect to private and public actors Law that Commandeers States (addressed only to the states) Whether or not this is ok depends on which branch is being commandeered If it is a policy making branch (legislative, and, according to Printz, the executive), the feds cant do it (barred by 10th) If it is not a policy making branch (judiciary), the feds can commandeer it, but the Pres and outside other outside sources cannot (Medellin) ICJ cant commander state judiciary, only commandeered if treaty is self-executing or if Congress has enacted the appropriate legislature Congress may not be able to escape the political heat for unpopular decisions but congress by careful use of enumerated powers can achieve practically any regulatory end it wants without running afoul of the Tenth Amendment. Could condition receipt of federal funds Directly regulate: For instance, in NY v. US could say no state may ship radioactive waste outside its borders.

Separation of Powers Part II: The Relationship Between Executive and Legislative Authority
Is the Proper Branch Acting? Justice Blacks Boxes / Formalistic Approach: What is the nature of the actor? What is the nature of his action? 13

If the actor is executive, the nature of the action must be executive, and so on. If the boxes dont line up, then the separation of powers doctrine prohibits the action. Presidential The President cant set policy. Youngstown The President cant tell the court what to do. Medellin v. Texas Congressional This generally isnt the problem with acts of Congress, but presentment and bicameralism are. Balancing Test Sometimes, the court merely employs a balancing act (see Morrison v. Olson) where is considers how great the intrusion is compared to how greatly we need it: To what extent does the act truly impede the other branchs power? Is the impediment culturally significant? Is it actually significant? Executive Authority: Presidential Power must stem from either the Constitution or Congress (delegation) Article I, Section 1 Confers the whole executive power to the president. Article II, Section 2 Lists all of the Presidents duties/powers. Inherent Powers: The President has certain inherent powers, such as the power to negotiate and the power to recognize foreign nations. Domestic Context Generally The Presidents power is not a generic go forth and run the country power. Commander in Chief does not refer to commander of the nation as a whole, but only of the armed forces. Justice Jacksons Levels of Presidential Power: Concurrence in Youngstown. Presidents powers are not fixed but fluctuate, depending on their disjunction or conjunction with those of Congress. 1) Highest: Congress has told him to act When the President acts pursuant to an express or implied authorization of Congress, his authority is at its maximum, for it includes all that he possess in his own right plus all that Congress can delegate Delegation Doctrine Congress may delegate to the Executive certain kinds of authority which belong to it, but which it wishes the Executive to utilize. 2) Middle: Congress is silent When the President acts in absence of either Congressional grant or denial of authority, he can only rely upon his own independent powers Twilight Zone There is a zone of twilight here where the President and Congress may have concurrent authority, or in which its distribution is uncertain. 3) Lowest: Congress disagrees When the President takes measures incompatible with the expressed or implied will of Congress, his power is at its lowest ebb, for then he can rely only upon his own constitutional powers, minus any constitutional powers of Congress over the matter. When Congress disapproves of what the President is doing, the President must hold the sole power to act. Dismissal Power The Pres can dismiss at any time those officers that serve at his will, except in the case of executive inferior officers whose dismissal is limited by Congress. What does at his will mean? If an officer has a solely executive function, then he serves at the will of the President. 14

However, if an officer as a semi-executive, semi-legislative role, he does not serve at the will of the Pres and Congress can set limits. It has nothing to do with who creates the position. For example, Congress creates the cabinet positions, but they serve at the will of the Pres. Presidential Immunities Testimonial Immunity Prosecuting crimes falls under the executives power, so generally the executive could grant himself immunity. This is not the case with Special Prosecutors, though, because all responsibility is delegated to them. If the Pres cannot use this immunity to get around testifying, he can go for executive privilege. However, the Court gets to determine when it can be invoked and it performs a balancing test, its a qualified privilege: Importance of getting info: Criminal > Civil > Congressional Hearings Importance in hiding info: military/diplomatic info > conversations relating to domestic governance > political conversations Seems like would be even more compelling if Defendant needed it for Due Process and since Pros can drop charges if dont want info released. Ct. not Executive decides whether privileged or not. US v. Nixon Nixon has to give up Watergate tapes. Civil Liability for Official Acts President has an ABSOLUTE IMMUNITY. Nixon v. Fitzgerald guy fired from criticizing military cost overruns and Ct. said for all acts within the outer perimeter of his authority. Immunity from Service of Process Immune from criminal service while sitting. President not immune from civil service as such cases would unduly distract him/occupy his time while serving. President is NOT immune from criminal liability for official acts. President is NEVER IMMUNE for the acts the President takes that are completely unrelated to the carrying out of his job. Clinton v. Jones Impeachment Article II 4 the President, Vice President and all civil Officers of the United States, shall be removed from Office on Impeachment for, and Conviction of Treason, Bribery, or other High Crimes and Misdemeanors. Two ways of viewing this: No all criminal actions are impeachable (drunk driving) Not all impeachable actions are criminal (lying under oath in a civil deposition) Ford: What constitutes an impeachable action is determined by a majority of Congress, what can get a president convicted is determined by 2/3 of the Senate Art. I 3, Cl.6: Majority of House 2/3 of Senate Its not a criminal conviction but can be convicted of crime after person is removed from office. Art.I 3, Cl.7. Question of what is a High crime or misdemeanor is prob not justiciable. See Nixon v. US (not pres Nixon). Foreign Context Article II, Sections 2 and 3 Provide that the Pres shall be Commander in Chief of the Army and Navy, that he has certain powers in regard to treaty-making, that he shall receive foreign ambassadors and ministers, and that he shall nominate and appoint US ambassadors and other public ministers, subject to Senate confirmation. Increased Power The President and Congress have a lot more authority under foreign affairs than under domestic, 15

because no power is shared with the states. They are held exclusively by the federal gov. US v. Curtiss Wright Curtiss Wright Upheld a joint resolution of Congress that authorized Prsident to ban the sale of to countries engaged in a particular conflict. Embargo extended to Bolivia. Congress can more easily delegate powers to the Pres in regards to international issues, because the decisions are a function of knowledge an confidentiality: the Pres is the most well situated to make these calls because he form treaties, negotiates, is head diplomat, etc. However, see Hamdan and Hamdi. While the Pres has a general military power as commander in chief, he can't simply circumvent Congress on military issues that still require Congressional approval. The Court wont allow it. Hamdan v. Rumsfeld Ct. rejected claim that Congress had implicitly granted (acquiesced) Bush power to set up a special category of military commissions to try Al Qaeda members for war crimes. (Pres speaks for the country, but not in all things.) Dames & Moore v. Regan Iran hostages case found support in prior legislation delegating power to pres. Congressional acquiescence silence as support. President has done something like this support and Congress never said anything so it is okay. So easier to find that Pres acted within his Scope.) Tried to limit scope saying this is not always case that Pres can suspend claims but need to look at whether suspension is a necessary incident to the resolution of a major foreign policy dispute. The Presidents power to control the military in the absence of a declaration of war is limited to defensive tactics and preparing for offense. However, given the recent events in Kosovo, Iraq, etc, this limit is being blurred. However, cannot take away fundamental rights like Habeas Corpus even if Congress agrees to let him. Boumediene v. Bush. (Guantanamo Habeas case) Executive Agreements Have the same authority as a treaty in regards to the Supremacy Clause. Usually require accompanying legislation (i.e. non-self-executing) the contrary is conceivable, but its hard to imagine what it would be Self-exec v. non Majority opinion in Medellin says look, real trick is what Cong intended when it ratified the treaty (and/or what pres intended when negotiated) Look at language / What other countries do with it / the context in which its passed (Vienna convention) Dissent look at nature of the acts to be undertaken If Judicial = self-exec If look leg = is non-self-exec Treaties Negotiated and signed by president / Ratified by 2/3 Senate Cannot violate external restraints (i.e. free speech) Dont have federalism concerns (i.e. like the CC does)

Legislative Authority: Domestic Context Old Non-Delegation Doctrine Congress cannot delegate its legislative authority to the President or to the states. If an Act is legislative, it has to comport with Article I. But then... Panama Refinding Co. Congress cannot delegate authority to the Pres without setting standards and guidelines. Schechter Poultry Congress cannot delegate to administrative agencies. 16

Now: These two cases are still good law, but the Court has not struck anything down as an excess of delegation since 1937. In contrast, the Court requires that Congresss delegation of powers contain standards to confine the discretion of the delegation. The standards can be lax good moral character (Chadha), public interest, convenience, necessity, etc. Presentment and Bicameralism Veto power: Article 1 7. Line Item Veto Congress gave the president the power to nix certain items on the budget, but pass the remainder and then used the money saved to reduce the budget (lock box provision). The Court found this unconstitutional for violating the Presentment Clause (Clinton v. City of NY). Scalia The name faked out the Court, because the President could equivalently decline to spend money after the budget was passed. Stewart agrees. The Court was possibly worried about political accountability on the part of Congress, because they would know that the Pres would just clean up their messes. Legislative Veto This allows Congress to give up a certain amount of responsibility, yet still retain power. They pass orders onto other branches, but permit one Congressional body the power to veto the other branchs decision. When Congress is acting in a legislative capacity, it must comply with bicameralism and presentment. The Legislative Veto violates both of these (INS v. Chadha). bicameral requirement of Article 1, 1 and 7. Usurpation Congress cannot usurp the executive branchs power. Congress cannot reserve for itself the power of removal of an officer charged with the execution of the laws except by impeachment. To permit an officer controlled by Congress to execute the laws would be to permit a congressional veto (Bowsher v. Synar). Appointment Power Article II Congress may be law vest the appointment of such inferior officers, as they think proper, in the Pres alone, in the Courts of law, or in the heads of departments (i.e. cabinet posts). Congress can limit the Pres ability to dismiss executive inferior officers so long as the removal restrictions are not of such a nature that they impede the Presidents ability to perform his constitutional duties. (Morrison v. Olson Ct. ruled statute was okay because the AG could terminate the special prosecutor for good cause). Article III, 1 Federal judges may not be removed except for misconduct. What are inferior officers? Not clear, but in Morrison it mattered to the Court that the officer was going to serve for a limited period of time and was answerable to the AG. Scalia thinks that inferior means subordinate, so an officer is inferior only if they serve directly under another. Foreign Context Article I, Section 8 Congress has the power to regulate commerce with foreign nations, to declare war, to raise and support armies, and to provide and maintain a navy. Congress enacted the War Powers Resolution to try to curb executive power and encourage further Congressional involvement, but it backfired. War Powers Resolution It attempted to balance two competing interests: (1) Presidents desire to be able to take armed forces anywhere in the world, and (2) Congresss desire to avoid interminable wars. Says the President can make decisions during war, but it is up to Congress to declare war. However, the President does have a 60 day window because of the need for immediacy in some instances, but after those 60 days, if Congress says no, the President must pull out. (This doesnt seem like it would be so easy to do.) 17

Equal Protection Two Governments, Two Clauses 14th Amendment Restricts state governments 5th Amendment Restricts the federal government defined by the S Ct to include an Equal Protection element that is the same as the 14th Classifications Which Distinguish Between Groups of People The Paradigm of Inequality: Race on the Constitution Constitutional Provision that Allude to Slavery: Article I, Section 2, Clause 3 whole number of free persons Article I, Section 9, Clause 1 Congress cant outlaw the importation of people Article IV, Section 2, Clause 3 required states to returned any person held to service or labor in one state who had escaped to that state State v. Post The NJ Court could have removed slavery from the political process in NJ by declaring that the state Constitution prohibited it, but it didnt. The line all men by nature are free did not abolish slavery. If the people had intended to abolish it, they would have explicitly stated so. Dred Scott The Court tried to take slavery out of the political process in order to settle the debate (clearly settled the wrong way). Whether or not the Court should do this is a huge issue. Held that black people were not citizens (slave or free) and limited the ability of the federal government to make territories truly free. Civil War, Reconstruction, Retreat Prior to the civil war, there were very few constraints on the power of state governments. Even the first eight amendments only protected individual rights from federal intrusion, not state intrusion. People viewed the states as protectors of their people and they feared federal domination. The Civil War changed this Before: The Constitution was viewed as a protection from federal power and the states were seen as a bulwark against federal interference. After: Constitution rights were seen as a basis for the assertion of federal power to protect individuals against state interference. The 13th, 14th, and 15th Amendments were passed in order to protect the newly freed slaves. The enabling clauses of the each were designed to give Congress broad power to enforce the amendments, especially the 14th. Courts Narrow Reading of the Reconstruction Amendments Slaughter-Hour Cases The Court didnt think that the 14th Am gave the fed gov the power to protect people in general, this led to a two-tiered system: 1) Rights of newly freed slaves are at stake? The amendment must be read expansively to provide comprehensive federal protection. Ex: Strauder v. West The Court reversed the murder conviction of a black man tried before a jury from which black people were excluded. 2) Racial discrimination not an issue? The protections of federal citizenship are narrower, a state residents primary recourse for protection of his rights remains with the state gov. Civil Rights Cases 18

The Court invalidated the public accommodations section of the 1875 Civil Rights Act, despite the 13th and 14th Amendments. 14th Am There must be inimical state action for a 14th Am violation. So, after the Slaughter-House Case and the Civil Rights Cases, in order to legislate under the 14th Am there had to be 1) an issue of race, and 2) state action that demonstrates animus with respect to the protected group of people. The lack of a state remedy to or prohibition of individual discrimination is not enough to allow Congress to step in and legislate. (Lack of law is not proof but its evidence if there is no recourse. The lawyers didnt make this argument, but it would probably hold water. Has anyone ever argued that a lack of remedy leads to state animus, or does Stewart just think it would work?) 13th Held that this amendment applied to both states and individuals and that Congress was empowered under the Am to pass all laws necessary and proper for abolishing all badges and incidents of slavery in the US. Housing discrimination didnt fall into this category a refusal of service had nothing to do with slavery or involuntary servitude. Plessy v. Ferguson Separate but equal is constitutional and not violative of the 14th Am, assuming the state is acting rationally (discouraging riots was considered rational). It was not constitutional, however, for states to enforce segregation against entities that did not want to be segregated. The legislatures can respond to the aggregated preferences of the people, but only the people who are affected (Berea College). This is an issue of race, and it involves state action, so why is it still ok under the SlaughterHouse Cases and the Civil Rights Cases? No animus on the part of the state There is no badge of inferiority because blacks are separate from whites just as whites are separated from blacks. Harlans Dissent: The constitutional is color-blind. The Constitution does not take into account a persons race when determining what their rights are. Chipping Away at Plessy Misouri ex rel. Gaines v. Canada The Court determined that it was up to them to determine the reasonableness of the segregated system de novo, which is a shift from Plesssy/Cummings. Sweatt v. Painter Held that the state had to admit a black student to the law school, because a comparable black institution would never be equal. Basically, in regards to law schools, Plessy could never be satisfied. Brown v. Board of Education The Court followed Harlans dissent in Plessy (the constitution is colorblind) and determined that race can NEVER be a deciding factor in state action. Even if something is equal and there is a legitimate government interest (i.e. a reasonable justification), it will still be unconstitutional if race is an issue. (Think about the black male schools set up in Detroit.) Courts Points: The only reason for the division was racial discrimination, and this can never be a legitimate governmental interest. The role of education has changes since Plessy It is now considered one of the critical things that states do (along with protect property interest, etc) and the 14th Am was designed to guarantee equality in these things. It instills in black a notion of inferiority (weak argument). Brown II 19

Remedy to school segregation: Admit to public schools on a racially nondiscriminatory basis with all deliberate speed. Problem: This language didnt force schools to desegregate immediately. The Court couldnt do more, though, because it didnt know if it could rely on the executive to enforce the order. What was tried in the South and didnt work: Freedom of Choice Plans Green v. County School Board found these to be unconstitutional because they were ineffective at achieving the goal in Brown an appropriate remedy must be an effective remedy Closing all Schools (and reopening as private) Constitutionally they could do this, but politically it would be a nightmare. Swann v. Charlotte-Mecklenburg Board of Education Determined that the constitution is colorblind only in regards to determining a constitutional violation. Race of student can be taken into account in order to remedy constitutional violations. This was adopted because the states in the South didnt comply with Brown II, and clearly a colorblind remedy wasnt effective. Must have de jure segregation! The fed ct. may not order de facto segregation, no matter how severe, to be cured by adjustment of racial balance or by rezoning. (school board itself could constitutionally exercise discretion to do this however.) Keyes v. School District No. 1 Once P can show that segregated schools have been brought about by intentional state action in a substantial portion of the schools, general desegregation is required throughout the system and P need not show deliberate segregation in each school. RULE: Must have a constitutional violation before being entitled to a constitutional remedy. To have a constitutional violation must have (1) Discriminatory Purpose and (2) Discriminatory Impact. Also recognized discrimination of Hispanics. Powell Concurrence/Dissent We should not distinguish between de facto (practical, residential housing patterns) and de jure (by law) segregation it is bad no matter why it exists and we should remedy it. But, he wants remedies that are less disruptive to the neighborhood school system (i.e. dislikes bussing). Four Steps the majority thinks this is ok Violation Remedy Unitary System Students Choosing Where to Go This is constitutional, assuming that the steps arent a violation themselves (i.e. imposed in order to segregate) Milliken v. Bradley You cant impose an inter-district remedy when there has been only an intra-district violation. The Court upheld the decision to resources to improve the black schools but to an extent this evokes separate but equal. US v. Fordice The Court found that dual university systems were unconstitutional where the state had failed to removed the incentives to stay segregated. Parents Involved Race as a classifier triggers stricter scrutiny because suspect trait suspect traits lead to heightened scrutiny, depending on which trait you are dealing with Acceptable Inequality How do we know which classifications are acceptable under the 14th Amendment Doctrinal Language? 20

Rational Basis Review: Lines are not based on any suspect classifications. Legitimate End + Rational Means Use cost/benefit analysis: Cost = What is lost to achieve the purpose; Benefit = Benefit the cost will bring. Court gives extreme deference to legislature under this review. Intermediate Scrutiny Review: Important End + Substantially Related Means Strict Scrutiny Review: Lines based upon suspect classifications (effects race or fundamental rights). Compelling State Interest + Necessary (Narrowly Tailored) Means Rational Basis Review 1) legitimate ends, 2) legitimate means, and 3) a rational relationship between the two This is a very differential standard. Assuming that the legislature has pointed to some legitimate purpose, it is very unlikely that the Court will determine that the means are irrational. (Federal legislation is invalidated more often, but this is due to federalism.) Over- Under-Inclusiveness The Court generally doesnt care if the statute is over or under inclusive, as long as long as the decision turns on the cost/benefit analysis and the effects of broader or narrower laws. NYC Transit v. Beazer Although this policy was both OVERINCLUSIVE (elminated some great employees from the applicant pool) and also UNDER-INCLUSIVE (did not eliminate alcoholics, child abusers, etc.), it was rational because the cost of screening was outweighed by the benefit of being able to exclude a group of people who you believe would be not very good employees. Ct. says there is a rational basis for such a policy. Questionably Matched Means and Ends The Court doesnt worry too much about whether there are more appropriate means. It also doesnt generally consider what the actual purpose of the statute was compared to the stated purpose. So long as there is at least one conceivable objective which is rationally related to the means selected, the possibility that another, illegitimate, objective might have motive the legislature will be ignored by congress. Legislation will not be invalidated only because dealt with part of the problem. Another way of saying that a statute which is under inclusive is not necessarily invalid. Railway Express Agency v. NY Regulation bans the placing of advertising on vehicles, except that the owner of a vehicle is permitted to advertise his own products. Williamson v. Lee Optical The Court upheld a law that prevented opticians from supplying new or duplicate lenses without a prescription. The Court held that the rule making body to remedy things however it wanted, provided there was no invidious discrimination. However, the Court goes back and forth on this. In Nordlinger v. Hahn, the Court suggested that it might be receptive to measuring a statute against the legislatures actual purpose. But it still said legit because there was a legit end of preserving the continuity and stability of local neighborhood by having higher taxes for newcomers. Real purpose prob to keep people out. Rational Wise If a rational legislature could have rationally believed that the ends fit the means, the Court wont touch it even if it turns out the legislature was dead wrong. 21

Minnesota v. Clover Leaf Creamery Co. The state allowed milk to be sold in paper containers, but not plastic, stating that the purpose was to help the environment. This proved to be false, but the Court wouldnt overturn it. That Court held that it didnt want to look Class of One It doesnt matter if only one person is disadvantaged she can still bring an equal protection claim. (Village of Willowbrook) Mere Rationality with Bite Sometimes the fit between the stated purpose and the means is so loose, and an alternative, illegitimate purpose is so obvious, that the Court cannot swallow it. Cases like this use the mere rationality language, but it is clear the Court is doing something more. This is generally employed when the Court thinks that the group being disadvantaged is politically unpopular. The Court looks to whether there was a bad actor and if the actual end was illegitimate. Ct. uses the following rationales: 1. the desire to harm an unpopular group cannot be a legitimate governmental object; or 2. that to the extend some apparently legit state objective is cited by the statutes defenders, the means drawn are so poorly linked to achievement of that objective that not even a rational relation between means and end is present. Moreno The food stamp requirements were invalidated because they were designed to negatively affect hippies, who are a politically unpopular group. Stated ends were (1) raise nutrition levels; and (2) help farmers. Post-hoc end was to deter fraud. Ct. allows this post-hoc end but means still not rationally related to end. Romer The Court invalidated an amendment to COs constitution that prevented local governments from protecting gay/lesbian/bi rights. The Court followed Moreno to invalidate it. Just harming a politically unpopular group and legit end of protecting other citizens freedom of association who object to homosexuality is not supported by the means. Way too loose. Scalias Dissent - Its ok to display animosity towards conduct that isnt constitutionally protected. The problem here is that you cant separate homosexual conduct from being homosexual. This discriminates against a group of people who are choosing to be who they are. You cant just remove from site a politically unpopular group. Cleburne The zoning restrictions didnt allow for houses for certain groups of people (insane, feeble-minded, alcoholics, etc). The city argued that it wanted to protect these people by keeping them isolated, but the Court said that you cant deal with harassment by eliminating the harassed. Suspect Classifications If the distinction in legislation is based on a suspect classification, the Court may heighten the standard of review. Gays (Romer), Mentally Retarded (Cleburne). Racial Classifications Disadvantaging Racial Minorities Racial classifications are inherently suspect and the Court will employ strict scrutiny. (Korematsu, Loving). Strict Scrutiny Requires: Compelling Ends Narrowly Tailored Means (Necessary and Narrowly Tailored, according to Korematsu) 22

Justifications for Using Strict Scrutiny 1) Race is generally irrelevant to a legitimate gov interest Discrimination is not a legitimate purpose (Loving). Racial prejudice of others cannot justify a governmental racial classification (Palmore v. Sidoti). The S Ct wont put its imprimatur on private discrimination. 2) History of slavery, the civil war, the 14th Am The aim of the 14th Am was against discrimination because of race or color (Strauder). 3) The idea of a caste system doesnt mesh with US ideals 4) Fundamental moral values equality of all people If this is one of our moral values, why do we do it? It is possibly unconscious prejudice, which means we need to fight it all the more. 5) Discrete (highly visible) & insular (isolated from political majority) minority that need to be protected In Hunter v. Erickson, the Court overturned a statute that required that housing statutes be approved by a majority of the electorate. The Court rejected it, stating that the laws impact falls on the minority. The majority needs no protection against discrimination and if it did, a referendum might be bothersome but no more than that. Such cases deal with raising the political hurdle. 6) Avoiding the we/them distinction Does legislation ever pass strict scrutiny? Yes, in Korematsu, the Court thought that pressing public necessity justified the law quarantining Japanese nationals. This has since been overturned. Also, see the school admissions cases. Non-Race-Specific Classifications that Disadvantage Racial Minorities In order for a non-race-specific law to trigger strict scrutiny, there must be purposeful discrimination / an intent to discriminate and a disparate impact (Washington v. Davis). Otherwise, only mere rationality applies. Knowledge Intent Knowledge that a minority will be burdened does NOT equal an intent to discriminate. Thus a law must be passed because of and not in spite of a disparate impact. Problem with this viewpoint: disparate impact reflects the reality of past constitutional violations, so strict scrutiny should always apply. This doesnt work, though, because at some point we want gov neutrality. Otherwise, the constitution will never be color blind. A D who alleges an EP violation has the burden of proving the existence of purposeful discrimination (McClesky). Wide Disparate Impact Will Not Change Requirement of Intent Consider McClesky The statistics showed that Georgias capital punishment statutes resulted in black people being much more likely to be sentenced to death than white people, especially if the black person killed a white person. Despite all of the evidence proving this, the man couldnt prove discriminatory intent and thus the Court did not apply SS. Even if he had proved intent, it would be hard to show that there was no compelling state interest and narrowly tailored means in the capital punishment statutes. This is the case of a statute with a neutral aim that is selectively indifferent to the welfare of certain groups. McClesky should have asked if the legislature would have been as willing to inflict the death penalty if most of the laws victims had been white. Dissent thought that the similarity between previous discriminatory laws + the statistics + probably of racial tension should have resulted in a different outcome. 23

Armstrong Statistics showed that 90% of crack convicts were black this Court didnt think this outweighed the intent requirement, D couldnt show that a white person would have been treated differently by the prosecutor NOTE: The two cases above deal with criminal law where the prosecutor has a lot of discretion, so you could argue for a different outcome in a civil case. To what do you look when determining purposeful discrimination (Village of Arlington Heights)? Different outcomes depending on race Bad acts/actors Impact of the official action Historical background of the decision Departures from normal procedures Substantive departures Legislative or administrative history Historical discrimination (Rogers v. Lodge) Particularly in cases like Rogers where the evidence shows that discriminatory practices were commonly utilized, that they were abandoned when enjoined by courts or made illegal by civil right legislation, and that they were replaced by laws and practices which, though neutral on their face, serve to maintain the status quo. Unfair Administration of Law Laws that are perfectly acceptable on their face, but that are administered unfairly, will trigger strict scrutiny (Yick Wo). What if the State proves it would have enacted the same law minus the animus? In Mt Healthy, a teacher argued that he had been fired for engaging in conduct protected by the first amendment. The Court reasoned that even though the teacher could prove the necessary facts, he wasnt automatically entitled to reinstatement if the school could prove by a preponderance of the evidence that it would have fired him anyway. This lowers the bar fundamental rights trigger strict scrutiny analysis, but preponderance of the evidence lines up with mere rationality. This has never been argued with EP, but if it were, and if the court followed this approach, it would change the analysis. It would be better to employ a higher burden of proof, possibly clear and convincing evidence. Proxy for Race Laws cannot use other traits as a proxy for race (Rice v. Cayetan). Note: Linguistic ability in choosing jurors is not a proxy for race because it is not the language ability but translational integrity that is at stake (Hernandez v. NY). Restructuring the Political Process Laws can place hurdles in front of everyone seeking to secure the benefits of governmental action. But when states allocate gov power non-neutrally, by explicitly using the racial nature of a decision to determine the decision-making process, strict scrutiny applies (Hunter, Seattle). In Washington v. Seattle, a state initiative prevented busing to other schools except in a few cases. Racial reassignment was one such case, but unlike the other exceptions, such reassignments were only allowed if a court found it to be constitutionally necessary. The Court thought that this placed an additional political hurdle in front of minorities (although, in reality, the hurdle was placed not in front of minorities, but in front of people who wanted to integrate so this only works if it is the minorities that want integration). Exception for Executive Decisions The Court has never told the Pres that he cant take race into account when making decisions. It is possible that it doesnt have to power to do this, but it is also possible that it is avoiding 24

the issue because it doesnt have the power to enforce it. Even if the Pres agreed, it would apply to so many things that it would be difficult to follow. Preemptory Challenges Preemptory challenges based on race are not allowed (Batson). However, Miller-El argued that this rule is unworkable and that the only way to get out this is to eliminate preemptory challenges all together. Affirmative Action (when the majority burdens the majority to benefit a minority) Laws that favor racial minorities are also subject to strict scrutiny review (City of Richmond v. J.A. Cronson), although initially the Court suggested that intermediate scrutiny was ok (Bakke). Reasons for Applying SS (Adarand) 1) Skepticism The Court must look closely as the laws to see which are invidious and which are benign. This doesnt mean fatal in fact OConnor recognizes that remedying past discrimination is a CI. 2) Consistency It doesnt matter who deals with the burden, it should be treated the same. The right to EP belongs to the individual, no to a race, so each should benefit equally from it. Stevens wanted to distinguish between invidious and benign laws, because it is politically less suspicious when the majority burdens itself. The problem with this is that it assumes that race is monolithic. But in any given case, it is very likely that there is a group of people (say white contractors) that are harmed by the laws and who were not represented in decision making process. 3) Congruency EP analysis should be the same whether it is under the 5th or 14th Am, this means no deferring to Congress. The Court doesnt want to defer because they are free from pressure and passion (counter-majoritarian branch). Whether or not this is necessary depends on how you perceive the race problem. If the problem is some species of a concern of a we/they divide (and race is the likely fault line given our history), then this role for the court makes sense. You need to keep a clear check on the use of race by any legislative body so as to no exacerbate the fault line. Any use of race underlines the fault line. But, if the real problem is some species of historical discrimination, than it is not the majority that needs the protection of the counter-majoritarian court, the minority does. So the court should offer some deference. Remedying Past Discrimination = Compelling Interest Congress Pursuant the 5 of 14th, Congress can enforce EP, so it is possible that the Court could give deference to congressional judgment in light of this power. However, Adarand precludes this possibility. Regardless, it is assumed that remedying past discrimination in general is a compelling interest for the Fed Gov (Fullilove), but ordinarily legislation is directed to a specific industry and hence to specific past acts of discrimination. States The States cannot claim that remedying general past societal discrimination is a compelling interest (Cronson). However, this does not go against Adarand. The reason for this restriction instead comes from Federalism. The state needs to focus on state issues, while the Fed Gov deals with things on a national scale. States can use past discrimination as a CI if the past discrimination is identified as a 25

constitutional or statutory violation (Cronson). Basically, it has to be very specific past discrimination you are remedying, with specific proof. Dissenting Views Scalia doesnt think that remedying past discrimination should ever be a CI. Thomas thinks that this is racially paternalist, which is just as pernicious as the initial discrimination. Student Diversity in Higher Education = Compelling Interest Quota Systems Dont Work This issue was first addressed in Bakke, where Powell cast the deciding vote in whether or not to uphold a quota system at the University of California. Powell thought that although the quota system violated EP, institutions of higher education do have a compelling interest in having a diverse class, and so schools may consider race as a plus among the factors they consider when making admissions decisions (Harvard Plan), but it cannot be determinative. Critical Mass Quota System It is unclear why these two arent the same, but the court has upheld the critical mass system (Grutter). (Harvard Plan just considers race as one factor, it is a +) Why is diversity a CI? Deep educational benefits: Enhances classroom discussion, promotes cross-cultural understanding, breaks down racial stereotypes for societal workforces, and allows us to effectively communicate across lines. The Court really deferred to the academics on this point, which is odd considering that they wont defer to Congress in regards to remedying past discrimination. Enhances classroom discussion, promotes cross-cultural understanding, breaks down racial stereotypes for societal workforces Dissents Scalia A few years in school is not going to change the way a person was raised, so this is a ridiculous assertion. Thomas This demeans the minorities. There are other ways the applicants could be chosen that would not take into account race. How are the means narrowly tailored? Race is not determinative, it is only a factor. It cannot be too large of a factor, though. In Gatz, all minorities were awarded 20 points on their applications just for being minorities. The Court thought this was determinative. It is a holistic approach. Dissent Rehnquist This is no different than a quota. NOTE: Kennedys dissent in Grutter suggests that AA plans are not valid unless there is a showing of past discrimination. NOTE: In Grutter, the Court held that racial considerations may not permanently be enshrined in admissions policies (25 years), but that periodic review of race-conscious admissions policies to determine whether racial preferences are still necessary to achieve student body diversity are sufficient to satisfy this limitation. Treatment of Native Americans Issues involving Native Americans are treated differently because there are remnants of sovereignty issues that dont exist in the context of other races/ethnicities. Gerrymandering in Order to Help Minorities The use of race in gerrymandering isnt necessarily unconstitutional, but it will trigger strict scrutiny (Miller). It can be a factor, but not the dominant purpose. This doesnt work if you disregard the traditional districting principals (compactness, 26

contiguity, and respect for political subdivision) in order to group members of one race together. It is insulting to suggest that members of a race all think alike (Shaw). Political gerrymandering is ok (Easley v. Cromartie), but how do you tell the difference between this and racial gerrymandering? Where minority-majority districts are at issue and where racial identification correlates highly with political affiliation, the party attacking the legislatively drawn boundaries must show at the least that the legislature could have achieved its legitimate political objectives in alternative ways that are comparably consistent with traditional districting principles. (Easley) Gender Classifications Gender issues are decided under an intermediate level of scrutiny, which requires (1) important ends and (2) substantially related means (Craig v. Boren). However, in one early case the Court did apply strict scrutiny (Frontiero). The justification for gender based governmental action must be genuine, not hypothesized or invented post hoc in response to litigation. And it must not rely on overbroad generalizations about the different talents, capacities, or preferences of males and females (US v. Virginia). Reasons for Heightened Scrutiny Gender is not chosen It is highly visible It is irrelevant to most government regulation There is a history of discrimination Reasons for Not Treating it the Same as Race Not a minority More political clout Women are at all levels of the economic scale (no we/they concerns) We are thus less of a monolithic group Everyone is exposed to an grows up with women We are not discrete and insular (discrete we are, but not discrete and insular) Real differences between the sexes (Nguyen) this usually ends up favoring women, though, see discussion of discrimination against men When the Court talks about this, it comes off as paternalistic. For example, in Nguyen, the Court assumed that the mother had a closer tie to the child because she raises it, where the father often doesnt. Potentially Valid Interest Remedying past gender discrimination. Kahn (Law granting property tax exemption to widows and not widowers furthers the state policy of cushioning the financial impact of spousal loss upon the sex for whom that loss usually imposes a heavier burden.) Califano v. Webster (SS act formula entitling female wage earners to a preference over males has been held to reduce the disparity in economic conditions between men and women, cause by the long history of discrimination against women.) Schlesinger (Navys mandatory discharge procedure, which accords different treatment for male and female line officers was upheld because the promotional opportunities offered by the Navy for men had been greater than for women.) Invalid State Interests Administrative ease does not support gender based discrimination (Reed). Upholding gender stereotypes is never an important interest, and the Court is hesitant to do it even if another interest is named (Boren). Old notions of sex roles are not sufficient justification for gender classifications (Weinberger, Stanton). 27

Even if a law is responding to the notion of different roles, the Court will not support laws that reinforce it (Hibbs Upheld the rule that both spouses must be able to take time off to act as the caregiver, because giving women more time reinforces the stereotype.) Single Sex Education The Court considered this in US v. Virginia, but it did not come up with any coherent answers. At the least, the educational opportunities offered to men and women must be evenhanded, but this always brings us back to Sweat. Rehnquist wrote separately in the VMI opinion suggesting that separate but equal schools wouldnt require the same curriculum, but instead simply must be of the same caliber and must not simply further gender stereotypes. The reason Rehnquist thinks that separate but equal works in the case of gender is that there are real differences between the sexes. Discrimination Against Men This is allowed when (1) Remedying past discrimination against women (see above), or (2) The state has a CI based upon real differences in the sexes Michael M v. Sonoma (Statutory rape laws prosecuting men but not women are ok, because preventing teen pregnancy is a CI and younger guy cant get pregnant.) Rostker (Requiring draft registration of males but not females, since females cant engage in combat. This deals partly with the Courts deference to the military power, but this might not fly today because of the way war has changed.) Nguyen (Upheld a fed statute that grants automatic US citizenship to nonmarital children born abroad to US mother, but requires that US fathers take specific steps to establish paternity in order to make such children US citizens see discussion above.) Would we be better with a bright line rule you cant make laws that are gender specific. It depends on whether, in the absence of discrimination against women, women are playing on a level playing field. This depends on whether you think the playing field is basically male (women are free to be men, but they cant be women, speaking with a different voice), or if women speaking in their very voice are able to change the norms on the playing field. Stewart doesnt think that there is any one answer for this. Related Issues Making conclusive assumptions based on gender violates Due Process (Stanley, LaFleur). The exclusion of women from juries deprives the D of his 6th Am right to a fair and impartial jury (Taylor). Sexual Orientation Classifications See someone elses notes. Did we really only talk about Romer? Romer never articulated the proper standard of scrutiny, but it seems like heightened scrutiny is appropriate. Why heightened scrutiny? Immutable trait History of discrimination Trait unrelated to ability to perform most tasks Why not strict scrutiny? Not a visible characteristic Due Process Amendments Restricting the Federal and State Governments 14th Am DPC restricts State Governments Nor shall any State deprive a person of life, liberty, or property, without due process of law... 28

5th Am DPC restricts the Fed Gov nor shall a person ... be deprived of life, liberty, or property without due process of law... What is Due Process and what does it protect? Procedural Due Process Just what it sounds like it places procedural limitations on the government when it deprives people of life, liberty, or property. Substantive Due Process What exactly is Substantive Due Process? It is a protection against arbitrary governmental action, because constitutionally unjustified laws are as arbitrary as are unconstitutional procedures. It seems that the states cannot violate fundamental rights with arbitrary legislation, and they act arbitrarily when they violate fundamental human rights, so its cyclical. The Due Process clause of the 14th Amendment is deemed to incorporate into the Am the protection of certain fundamental rights. This trend of incorporation started with the Slaughter-House Cases, when the court noted that provisions of the Bill of Rights may be sufficiently fundamental as to be protected against state abridgement through the 14th Am DPC. Did we talk about/read this part of the SHC? Why would the court do this here? What is a fundamental right? That depends on how you look at it. There are two competing views, but generally a good starting place is to look at the Bill of Rights. Things that are necessary to the concept of ordered liberty, OR Fundamental principles of Anglo-American justice. Not all rights enumerated in the Bill of Rights have been incorporated into the 14th Am, but most of the biggies have been (freedom of speech, religion, etc). There was and continues to be a huge debate about whether there are fundamental rights untethered to the first eight amendments. While Griswold did indeed speak in terms on penumbras and emanations, later cases dont and some of the concurring opinions in that case saw the existence of fundamental rights other than those there listed as supported by the 9th or the 14th Amendment. Why was it necessary to incorporate the Bill of Rights into the 14th Am? The Bill of rights was only enforceable against the Fed Gov, not the states, and this needed to be changed (due to the realities of slavery, etc). When Congress introduced the 14th Am, included in it was the Privileges & Immunities Clause that held: No States shall make or enforce any law which shall abridge the privileges or immunities of citizens of the United States... The P&I Clause seems to indicate that at least certain fundamental rights would be enforceable against the states, but the Court ruled that the guarantees in the Bill of Rights were not privileges and immunities of national citizenship protected from abridgment by the P&I C (Slaughter-House Cases), effectively writing the P&IC out of the constitution. NOTE: There is no incorporation issue with the 5th Amendment because its not necessary the Bill of Rights is already applicable to the Fed Gov. Economic Rights as Fundamental and Protected from Government Regulation Lochner Era Approach The Court indicated that certain fundamental rights exist even if they are not specifically mentioned in the constitution. This included economic rights that were encompassed by the protection of liberty and property in the DPC. In Lochner, the Court dealt with the right to enter into Ks (an economic right). It held that, because it was a fundamental right, it could not be arbitrarily regulated. What is Arbitrary Regulation? Regulation that lacks either a legitimate ends or rationally related means (real and substantial means) so basically, it must pass mere rationality review. The Court claimed that protecting the public health, safety, welfare, or morals was a 29

legitimate end, but this did not fit the bill (see next point). Courts Level of Deference? The Court thought that it was basically a labor law, and the Court at that time thought that this could never be a legitimate purpose. Thus they did not defer to Congress. Instead, the insisted on upholding the laissez faire market place (even though, as the dissent noted, the Constitution doesnt chose an economic theory). Modern Approach The Court no longer views economic rights as fundamental, due in large part to the great depression and failure of the laissez faire economy. Now, the Gov can regulate more freely and the Court generally defers to Congress. Level of Deference It is extreme deference. Even if it turns out that Congress didnt do its homework at all, the Court wont overturn the legislation so long as Congress could rationally have thought that the ban could serve a legitimate state interest (see Carolene Products). Modern Notions of Fundamental Rights The Court returned to a Lochner-esque mindset in regards to finding certain things to be fundamental rights, even though they are not explicitly mentioned in the constitution. However, the court did NOT revive Lochner for two reasons: Lochner set a low standard of review (rational review basis) and the justices in these cases employed strict scrutiny. The idea of economic rights as fundamental had gone by the wayside. Privacy Early on the Court suggested that the right to procreate is a fundamental right (one of the basic civil rights of man), but it did not consider it under the DPC and thus did not have to justify why it was a fundamental right (Skinner v. Oklahoma). The violation of this fundamental right was subject to strict scrutiny under the EPC. Why was this fundamental? The Court mentions that it is necessary for the survival of the race, but this is mostly a make weight argument. More than anything, it would seem that this is a fundamental right because it involves the ability of the felon to define himself (as a father). Procreation is essential to self-definition. Where does the right to privacy come from? Different justices have justified it in different ways: Penumbras or Emanations of Privacy Case law regarding the First Am suggests that specific guarantees in the Bill of Rights have penumbras, formed by emanations from those guarantees that help give them life and substance. This creates a zone of privacy where to gov cannot regulate (Griswold). Only fundamental rights listed in the Constitution create these penumbras. For example, there is no penumbra around the right to K, because it is no longer considered a fund right. Ninth Amendment The enumeration of certain rights shall not be construed to deny or disparage others retained by the people... (Griswold, Goldbergs Concurrence). According to Goldberg, these rights are the traditions and the collective conscious of our people ... so rooted there as to be ranked as fundamental. Problem: This presupposes that we have a collective conscious. Implicit in the Concept of Liberty Harlans Concurrence first mentions this in Griswold (the liberty mentioned in the 14th Am stands on its own). 30

Thought that fund rights were the rights that which belong to citizens of all free governments. Roe v. Wade appears to follow this reasoning later on. Personal rights that can be deemed fundamental or implicit in the concept of ordered liberty, are included in this guarantee of personal liberty. Notice that none of the three justifications mention Due Process. It is because the Court was afraid to rely on it after Lochner. Specific rights that have been protected: Marriage (Loving overturned the miscegenation laws) Procreation (Skinner overturned the law allowing for the sterilization of convicts) The notion of self-identification Contraception (Griswold you cant bar the use of contraceptives; Eisenstadt you cant limit the sale to certain people) The notions of autonomy and the personal choice not to have kids (relates to self-expression); self-determination The first case deals with the zone of marital privacy, the second deals with individual privacy. Abortion (Roe, Casey see below) The Decision to Bear Children What Supports this Right? When the Court upheld the right of a woman to get an abortion, it did not clearly articulate where the right came from (autonomous decision making, penumbras of privacy, implicit in liberty, or otherwise). However, it at least appears that the court was supporting the notion of privacy in regards to bodily integrity and not just in the sense of personal choice / autonomy, as distinguished from previous cases (Roe). What Made Roe Difficult? Even though it is reviewed under strict scrutiny, the state has two compelling interests which are at odds with each other and with the womans right to privacy. Interest in protecting the fetus. Interest in protecting the mothers health. Result First Trimester: No state regulation Second Trimester: State can regulate to protect the mothers health Third Trimester: State can regulate to protect the unborn child, it can even prohibit abortion, except in cases where the mothers health is in jeopardy (interest in mother is greater than that in child) Limits on Roe Shift to a Paternalistic Approach Funding The Court has held that the state was not required to fund abortions. Had Roe been decided on equal protection grounds, this case may have been different (Maher). Abandonment Strict Scrutiny In Casey, OConnor argued to affirm Roe for four reasons (stare decisis, detrimental reliance, the workability of the doctrine, and the fact that the court was left with more than remnants of an abandoned doctrine). She also agreed that the woman had a liberty interest in being able to choose, so overall it would seem that the right to choose is still fundamental. However, she did not employ strict scrutiny to the law in question, instead she laid out the undue burden test which allows the state to regulate in the first trimester even though there is no compelling interest identified. New Viability Test: Pre-Viability The states interest in protecting the womans health and the fetus are 31

outweighed by the womans right to have an abortion without imposition of undue burdens. Undue burden = something that places a substantial obstacle in front of the woman who wants to terminate a pregnancy Post-Viability The state may prohibit abortion except when necessary to reserve the mothers life. 24 hour waiting periods and providing the woman with information to persuade her otherwise were NOT deemed to be undue burdens (Casey). Requiring that the woman inform her has WAS deemed an undue burden (places unfair burden on women who are mentally/emotionally abused or whose children may be abused). You limit to the people who it burdens to determine if it is an undue burden. Partial Birth Abortions Generally, a ban on PBAs is ok as long as there is an exception for the health of the mother. From Abortion to Infanticide In regards to partial birth abortions, the Court held that whether it was an abortion or infanticide no longer depended on viability, but on whether the baby was outside of the womb (Gonzalez v. Carhart). Protecting the Woman From Herself as a Legitimate Interest Kennedy stated that promoting respect for human life is a legitimate interest, and that Respect for life finds an ultimate expression in the bond of love the mother has for her child ... Whether to have an abortion requires a difficult and painful moral decision. While we find no reliable data to measure the phenomenon, it seems unexceptional to conclude some women come to regret their choice to abort the infant life they once created and sustained. Severe depression and loss of esteem can follow. (Gonzalez)

Family The right to privacy encompasses familial choices as well (this has been posited in Skinner, Griswold, Roe, etc.), and the Court is willing to protect the family. Hierarchy: Nuclear Family (i.e. husband, wife, children) The Court will protect the NF against both governmental intrusions and intrusions by extended family members. Exception: Laws who tell who/when we can marry. Can be conditioned on age, consanguinity, bigamy, but not miscegenation. Extended Family The Court is willing to protect the extended family against governmental intrusion (Moore). Homosexuality Originally, the court didnt think that the right to privacy extended to the right to engage in homosexual sodomy (Bowers). This opinion was strongly rooted in the traditional/historical concept of one man one woman as a family. Problem: This presupposes that marriage is natural law and not a social construct, which is not the case. When viewed in the light of social construct, the homosexual sodomy is wrong argument doesnt work. In the Courts defense, the recognition of homosexuality is a relatively new social construct. Now, the Court has abandoned Bowers, stating that the liberty protected by the Constitution allows homosexual persons the right to make this choice (Lawrence). However, the Court never states that it is a fundamental right. This goes back to physical autonomy. The Court didnt apply strict scrutiny. They applied mere rationality with bite and went back to Moreno. Right to Die Everyone has the right to refuse medical treatment (Cruzan). It is not treated as suicide, because it is an action of omission as opposed to commission. 32

If the patient makes no choice, it is ok for the state to set the default at the preservation of life, regardless of what the family wants (Cruzan). However, a few justices look at this through the guise of family autonomy and suggest that the family should be the one to decide. There is no liberty interest in suicide, thus it is not a fundamental right and the Court need not apply strict scrutiny (Glucksberg). The state has a legitimate interest in protecting life, and thus doctor assisted suicides can be regulated / prohibited. At least three Justices think that if the purpose of medication is pain relief, it doesnt matter than one of the side effects is death.

33

Вам также может понравиться